Derivative of $x^{beta}$












1












$begingroup$


Let $xinmathbb{R}^d$, and $alpha,betainmathbb{N}^d$ such that $|alpha|inleft{1,2right}$ $|beta|ge 3$



What is the result of $$partial_x^{alpha}x^{beta}$$?



Thanks










share|cite|improve this question











$endgroup$












  • $begingroup$
    So $alpha$ and $beta$ are vectors whose components can only be natural numbers (in addition to the other restrictions)?
    $endgroup$
    – Chase Ryan Taylor
    Dec 3 '18 at 20:21










  • $begingroup$
    yes that is it exactly
    $endgroup$
    – aymen
    Dec 3 '18 at 20:21
















1












$begingroup$


Let $xinmathbb{R}^d$, and $alpha,betainmathbb{N}^d$ such that $|alpha|inleft{1,2right}$ $|beta|ge 3$



What is the result of $$partial_x^{alpha}x^{beta}$$?



Thanks










share|cite|improve this question











$endgroup$












  • $begingroup$
    So $alpha$ and $beta$ are vectors whose components can only be natural numbers (in addition to the other restrictions)?
    $endgroup$
    – Chase Ryan Taylor
    Dec 3 '18 at 20:21










  • $begingroup$
    yes that is it exactly
    $endgroup$
    – aymen
    Dec 3 '18 at 20:21














1












1








1





$begingroup$


Let $xinmathbb{R}^d$, and $alpha,betainmathbb{N}^d$ such that $|alpha|inleft{1,2right}$ $|beta|ge 3$



What is the result of $$partial_x^{alpha}x^{beta}$$?



Thanks










share|cite|improve this question











$endgroup$




Let $xinmathbb{R}^d$, and $alpha,betainmathbb{N}^d$ such that $|alpha|inleft{1,2right}$ $|beta|ge 3$



What is the result of $$partial_x^{alpha}x^{beta}$$?



Thanks







real-analysis partial-derivative






share|cite|improve this question















share|cite|improve this question













share|cite|improve this question




share|cite|improve this question








edited Dec 4 '18 at 11:10









Harry49

6,17331132




6,17331132










asked Dec 3 '18 at 20:06









aymenaymen

304




304












  • $begingroup$
    So $alpha$ and $beta$ are vectors whose components can only be natural numbers (in addition to the other restrictions)?
    $endgroup$
    – Chase Ryan Taylor
    Dec 3 '18 at 20:21










  • $begingroup$
    yes that is it exactly
    $endgroup$
    – aymen
    Dec 3 '18 at 20:21


















  • $begingroup$
    So $alpha$ and $beta$ are vectors whose components can only be natural numbers (in addition to the other restrictions)?
    $endgroup$
    – Chase Ryan Taylor
    Dec 3 '18 at 20:21










  • $begingroup$
    yes that is it exactly
    $endgroup$
    – aymen
    Dec 3 '18 at 20:21
















$begingroup$
So $alpha$ and $beta$ are vectors whose components can only be natural numbers (in addition to the other restrictions)?
$endgroup$
– Chase Ryan Taylor
Dec 3 '18 at 20:21




$begingroup$
So $alpha$ and $beta$ are vectors whose components can only be natural numbers (in addition to the other restrictions)?
$endgroup$
– Chase Ryan Taylor
Dec 3 '18 at 20:21












$begingroup$
yes that is it exactly
$endgroup$
– aymen
Dec 3 '18 at 20:21




$begingroup$
yes that is it exactly
$endgroup$
– aymen
Dec 3 '18 at 20:21










1 Answer
1






active

oldest

votes


















1












$begingroup$

Let $alpha=(alpha_1,dots,alpha_n)$ and $beta=(beta_1,dots,beta_n)$. If $alphalebeta$, that is, $alpha_klebeta_k$, $1le kle n$,Then
begin{align}
partial_x^alpha x^beta&=prod_{k=1}^nfrac{partial^{alpha_k} x_k^{beta_k}}{partial x_k^{alpha_k}}\
&=prod_{k=1}^nbeta_k(beta_k-1)dots(beta_k-alpha_k+1)x_k^{beta_k-alpha_k}\
&=Bigl(prod_{k=1}^nfrac{beta_k!}{(beta_k-alpha_k)!}Bigr)x^{beta-alpha}\
&=frac{beta!}{(beta-alpha)!},x^{beta-alpha}.
end{align}






share|cite|improve this answer









$endgroup$













    Your Answer





    StackExchange.ifUsing("editor", function () {
    return StackExchange.using("mathjaxEditing", function () {
    StackExchange.MarkdownEditor.creationCallbacks.add(function (editor, postfix) {
    StackExchange.mathjaxEditing.prepareWmdForMathJax(editor, postfix, [["$", "$"], ["\\(","\\)"]]);
    });
    });
    }, "mathjax-editing");

    StackExchange.ready(function() {
    var channelOptions = {
    tags: "".split(" "),
    id: "69"
    };
    initTagRenderer("".split(" "), "".split(" "), channelOptions);

    StackExchange.using("externalEditor", function() {
    // Have to fire editor after snippets, if snippets enabled
    if (StackExchange.settings.snippets.snippetsEnabled) {
    StackExchange.using("snippets", function() {
    createEditor();
    });
    }
    else {
    createEditor();
    }
    });

    function createEditor() {
    StackExchange.prepareEditor({
    heartbeatType: 'answer',
    autoActivateHeartbeat: false,
    convertImagesToLinks: true,
    noModals: true,
    showLowRepImageUploadWarning: true,
    reputationToPostImages: 10,
    bindNavPrevention: true,
    postfix: "",
    imageUploader: {
    brandingHtml: "Powered by u003ca class="icon-imgur-white" href="https://imgur.com/"u003eu003c/au003e",
    contentPolicyHtml: "User contributions licensed under u003ca href="https://creativecommons.org/licenses/by-sa/3.0/"u003ecc by-sa 3.0 with attribution requiredu003c/au003e u003ca href="https://stackoverflow.com/legal/content-policy"u003e(content policy)u003c/au003e",
    allowUrls: true
    },
    noCode: true, onDemand: true,
    discardSelector: ".discard-answer"
    ,immediatelyShowMarkdownHelp:true
    });


    }
    });














    draft saved

    draft discarded


















    StackExchange.ready(
    function () {
    StackExchange.openid.initPostLogin('.new-post-login', 'https%3a%2f%2fmath.stackexchange.com%2fquestions%2f3024589%2fderivative-of-x-beta%23new-answer', 'question_page');
    }
    );

    Post as a guest















    Required, but never shown

























    1 Answer
    1






    active

    oldest

    votes








    1 Answer
    1






    active

    oldest

    votes









    active

    oldest

    votes






    active

    oldest

    votes









    1












    $begingroup$

    Let $alpha=(alpha_1,dots,alpha_n)$ and $beta=(beta_1,dots,beta_n)$. If $alphalebeta$, that is, $alpha_klebeta_k$, $1le kle n$,Then
    begin{align}
    partial_x^alpha x^beta&=prod_{k=1}^nfrac{partial^{alpha_k} x_k^{beta_k}}{partial x_k^{alpha_k}}\
    &=prod_{k=1}^nbeta_k(beta_k-1)dots(beta_k-alpha_k+1)x_k^{beta_k-alpha_k}\
    &=Bigl(prod_{k=1}^nfrac{beta_k!}{(beta_k-alpha_k)!}Bigr)x^{beta-alpha}\
    &=frac{beta!}{(beta-alpha)!},x^{beta-alpha}.
    end{align}






    share|cite|improve this answer









    $endgroup$


















      1












      $begingroup$

      Let $alpha=(alpha_1,dots,alpha_n)$ and $beta=(beta_1,dots,beta_n)$. If $alphalebeta$, that is, $alpha_klebeta_k$, $1le kle n$,Then
      begin{align}
      partial_x^alpha x^beta&=prod_{k=1}^nfrac{partial^{alpha_k} x_k^{beta_k}}{partial x_k^{alpha_k}}\
      &=prod_{k=1}^nbeta_k(beta_k-1)dots(beta_k-alpha_k+1)x_k^{beta_k-alpha_k}\
      &=Bigl(prod_{k=1}^nfrac{beta_k!}{(beta_k-alpha_k)!}Bigr)x^{beta-alpha}\
      &=frac{beta!}{(beta-alpha)!},x^{beta-alpha}.
      end{align}






      share|cite|improve this answer









      $endgroup$
















        1












        1








        1





        $begingroup$

        Let $alpha=(alpha_1,dots,alpha_n)$ and $beta=(beta_1,dots,beta_n)$. If $alphalebeta$, that is, $alpha_klebeta_k$, $1le kle n$,Then
        begin{align}
        partial_x^alpha x^beta&=prod_{k=1}^nfrac{partial^{alpha_k} x_k^{beta_k}}{partial x_k^{alpha_k}}\
        &=prod_{k=1}^nbeta_k(beta_k-1)dots(beta_k-alpha_k+1)x_k^{beta_k-alpha_k}\
        &=Bigl(prod_{k=1}^nfrac{beta_k!}{(beta_k-alpha_k)!}Bigr)x^{beta-alpha}\
        &=frac{beta!}{(beta-alpha)!},x^{beta-alpha}.
        end{align}






        share|cite|improve this answer









        $endgroup$



        Let $alpha=(alpha_1,dots,alpha_n)$ and $beta=(beta_1,dots,beta_n)$. If $alphalebeta$, that is, $alpha_klebeta_k$, $1le kle n$,Then
        begin{align}
        partial_x^alpha x^beta&=prod_{k=1}^nfrac{partial^{alpha_k} x_k^{beta_k}}{partial x_k^{alpha_k}}\
        &=prod_{k=1}^nbeta_k(beta_k-1)dots(beta_k-alpha_k+1)x_k^{beta_k-alpha_k}\
        &=Bigl(prod_{k=1}^nfrac{beta_k!}{(beta_k-alpha_k)!}Bigr)x^{beta-alpha}\
        &=frac{beta!}{(beta-alpha)!},x^{beta-alpha}.
        end{align}







        share|cite|improve this answer












        share|cite|improve this answer



        share|cite|improve this answer










        answered Dec 4 '18 at 15:40









        Julián AguirreJulián Aguirre

        68.4k24094




        68.4k24094






























            draft saved

            draft discarded




















































            Thanks for contributing an answer to Mathematics Stack Exchange!


            • Please be sure to answer the question. Provide details and share your research!

            But avoid



            • Asking for help, clarification, or responding to other answers.

            • Making statements based on opinion; back them up with references or personal experience.


            Use MathJax to format equations. MathJax reference.


            To learn more, see our tips on writing great answers.




            draft saved


            draft discarded














            StackExchange.ready(
            function () {
            StackExchange.openid.initPostLogin('.new-post-login', 'https%3a%2f%2fmath.stackexchange.com%2fquestions%2f3024589%2fderivative-of-x-beta%23new-answer', 'question_page');
            }
            );

            Post as a guest















            Required, but never shown





















































            Required, but never shown














            Required, but never shown












            Required, but never shown







            Required, but never shown

































            Required, but never shown














            Required, but never shown












            Required, but never shown







            Required, but never shown







            Popular posts from this blog

            Plaza Victoria

            In PowerPoint, is there a keyboard shortcut for bulleted / numbered list?

            How to put 3 figures in Latex with 2 figures side by side and 1 below these side by side images but in...